LSAT and Law School Admissions Forum

Get expert LSAT preparation and law school admissions advice from PowerScore Test Preparation.

 Administrator
PowerScore Staff
  • PowerScore Staff
  • Posts: 8919
  • Joined: Feb 02, 2011
|
#41633
Complete Question Explanation
(The complete setup for this game can be found here: lsat/viewtopic.php?t=2813)

The correct answer choice is (D)

If T is presented fifth, we need to work with Template 1 in determining the order of the remaining variables (no other solution allows for T to be fifth). Since T must be presented earlier than W, it follows that W must be presented sixth:
PT65_D11 LG Explanations_game_#2_#7_diagram 1.png
The second and fourth presentations must therefore be given by J, establishing the following dual options:
PT65_D11 LG Explanations_game_#2_#7_diagram 2.png
If you decided not to make templates, you can solve this question as follows: recall that T is presented by K, who cannot give any two presentations in a row (first rule). If K presents fifth, the Separation Principle requires K to also present first and third, in compliance with the first rule:

..... ..... ..... ..... ..... ..... ..... 1 - 3 - 5

This immediately eliminates answer choices (B) and (C). It also validates answer choice (D), because S is presented by K, and can therefore be presented third.

Answer choice (A) is incorrect, because T must be presented before W. If T is presented fifth, then W must be presented sixth.

Answer choice (B) is incorrect, because P is presented by K, and must therefore be presented either first or third.

Answer choice (C) is incorrect, because S is presented by K, and must therefore be presented either first or third.

Answer choice (D) is the correct answer choice, as explained above.

Answer choice (E) is incorrect, because W must be presented last.

Get the most out of your LSAT Prep Plus subscription.

Analyze and track your performance with our Testing and Analytics Package.